Derivación más simple de la ecuación de Sackur-Tetrode

¿Hay alguna razón por la que la siguiente derivación de la ecuación de Sackur-Tetrode no sea común? Estoy enseñando una clase de nivel de pregrado inferior y me gustaría derivarla con términos más simples de solo usar volumen relativo.

la ecuacion es:

k norte en V norte λ 3 ,
dónde λ 3 es la longitud de onda térmica al cubo, o el volumen cuántico de una partícula.

Como cada partícula tiene un volumen de λ 3 , el total del número de posiciones en el volumen de una partícula es norte = V λ 3 , y norte es el número total de partículas.

  1. Usando la distribución binomial, la definición de S de la ecuación de Boltzmann es:
    S = k en Ω = k en [ norte ! norte ! ( norte norte ) ! ]
  2. reemplazando norte = V λ 3 ,
    S = k en [ ( V λ 3 ) ! norte ! ( V λ 3 norte ) ! ]
  3. Usando la aproximación de Stirling:
    S = k [ V λ 3 en ( V λ 3 ) ( V λ 3 norte ) en ( V λ 3 norte ) norte en ( norte ) ]
  4. Usando la aproximación en ( V λ 3 norte ) = en ( V λ 3 ) para V λ 3 norte
    S = k [ norte en ( V λ 3 ) norte en ( norte ) ]
  5. Manipular álgebra.
    S = k norte en V norte λ 3

Me gustaría usar esto en mi clase porque es más simple y limpio y desarrolla una especie de intuición química basada en la distribución binomial. Sin embargo, quiero saber si esto es correcto. He visto esta ecuación sin el 5 / 2 término antes.

Editar: la suposición física que creo que podría no ser válida es el uso de N como V λ 3 . La distribución binomial es válida cuando tienes n partículas que caben en N posiciones DISCRETAS. Eso funcionaría bien para una caja de volumen V con N ranuras separadas por particiones. Pero en este caso, el volumen V no tiene particiones y es continuo. En este caso, un volumen λ 3 puede encajar en posiciones que no están discretamente definidas. Esto crea un número técnicamente infinito de N. Entonces, ¿sería esto todavía válido? Ω para ser utilizado en la ecuación de Boltzmann?

Hola Ted, Mathjax se usa en este sitio para expresiones como las anteriores y, dado que hace que las posibles respuestas sean más probables, en mi opinión, podría valer la pena realizar una edición rápida. Mucha suerte con tu pregunta.
Gracias por el consejo. Acabo de aprender Mathjax, que fue bastante fácil. Con suerte, algunas personas responderán.
Además, creo que las personas que respondan a sus preguntas pueden copiar y pegar su arduo trabajo, lo que aumenta las posibilidades de una respuesta.
La gente está votando para poner su pregunta en espera, lo que puede deberse a que no está muy claro por qué sospecha que esta derivación puede no ser correcta. Si pudiera ampliar eso, creo que mejoraría la pregunta considerablemente. Tendemos a ser duros con las preguntas que solo nos piden que verifiquemos algo, en parte como una medida para desalentar a las personas que buscan ayuda con la tarea.
PD: arreglé el formato matemático para ti.
Sé que las matemáticas son correctas. Estoy planeando usar esta derivación en mi chem e. clase de termodinámica en dos semanas. Hice esta derivación yo mismo. Sin embargo, dado que no he visto esta derivación en ninguna parte de la web (incluido este sitio web) o libros de texto, quiero saber por qué. ¿Lo configuré de una manera que viola las leyes de la física? Además, si alguien ve esta derivación en otro lugar, también sería genial. Es una derivación tan simple en comparación con las demás, que tiene que haber una razón por la que no se usa con más frecuencia.
Realmente no entiendo este sitio. ¿¿La razón para poner esto en espera es porque sospechas que necesito ayuda con una pregunta de tarea? Aquí está mi sitio web y el horario de clases para la clase de termodinámica que estoy enseñando.
Simplemente no entiendo el sistema de revisión aquí. Por lo general, debe haber algún tipo de revisión por pares en la que los revisores planteen sus preocupaciones y haya algún tipo de refutación. Si tiene inquietudes, envíeme un correo electrónico a ted.yu@csulb.edu y resolvamos esto profesionalmente. Esta es la primera vez que uso este sitio, y la experiencia hasta ahora no ha sido buena, excepto por la buena respuesta de David Z y Acid Jazz.
Estoy de acuerdo en que esta no es una pregunta del tipo "Por favor, deriva esto para mí" que consideramos fuera de tema ; en cambio, está preguntando "¿Por qué no es esto común?", lo que lo hace fuera de tema, ya que está pidiendo opiniones sobre algo.
Si aún no lo ha hecho, eche un vistazo al Centro de ayuda , ya que puede explicar el modelo SE mucho mejor que yo en un cuadro de comentarios de ~500 caracteres.
Ted Yu - Mi voto para cerrar (que por casualidad resultó ser el último) se debió a que, en la(s) versión(es) inicial(es) (hasta la revisión 6), se trataba simplemente de una derivación, que en este caso es tarea -como, aunque no es un problema de tarea real. La versión actual (revisión 7) deja más claro que en realidad se trata solo del concepto involucrado, por lo que votaré para reabrir.
Gracias por el voto (o desvoto) de confianza HDE. Sin embargo, su voto aún no abre esta pregunta.
Kyle: La pregunta no es por qué esto no es común, sino ¿es válida la derivación? La suposición que hace la gente es que es correcta, simplemente porque la respuesta es correcta. Una derivación más simple puede ahorrar mucho tiempo de clase, así como tiempo para que los estudiantes comprendan un concepto. Pero tengo cuidado de no introducir una derivación simple si tiene fallas. Ahora entiendo por qué esta placa tiene problemas con esta pregunta, porque la mayoría simplemente asume que la derivación es correcta debido a la respuesta correcta. En cierto modo, estás tomando atajos como la persona que viene aquí en busca de respuestas gratuitas.

Respuestas (1)

Problemas con esa derivación:

  • Te estás perdiendo el término adicional 5 2 k norte , lo que puede ser importante si tiene que hacer algún trabajo con potenciales químicos.
  • Sus alumnos no necesariamente sabrán por qué parcelar el espacio en volúmenes de tamaño λ 3 . Partiendo de la definición de entropía y deduciendo que el volumen térmico λ 3 es importante parece preferible.
  • Sus estudiantes pueden beneficiarse al saber que los semifactoriales existen a través de norte ! = 0 d X   X norte   mi X , que parten de ( 1 / 2 ) ! = π , y que el volumen de un norte -bola de radio r es π norte / 2   r norte / ( norte / 2 ) ! . Son solo unos quince minutos o menos en una pizarra de trucos matemáticos, pero puede ayudar a facilitar algunos cálculos posteriores, además de un "oh, así es como funciona" que explica por qué los gaussianos tienen estos misteriosos factores de π en ellos.
  • El hecho de que la mecánica cuántica haga que las partículas sean incluso en principio indistinguibles es enorme y resuelve la paradoja de Gibbs, lo que lleva a la ecuación de Sackur-Tetrode. Robarle a un estudiante este hecho, y la cantidad de distorsión mental que crea, no es necesariamente algo malo , pero tampoco es necesariamente algo bueno . Esta es una oportunidad para plantar una semilla de disonancia cognitiva que solo se resolverá realmente cuando el estudiante aprenda la teoría cuántica de campos. Es quizás incluso el problema central con la comprensión del mundo centrada en partículas.
  • Si está enseñando mecánica estadística, sus alumnos apenas comienzan a comprender qué es realmente la temperatura. No estoy seguro de que personalmente lo convertiría en una "viga de carga" de su comprensión de la mecánica estadística. Para aprender termodinámica antes de un curso de mecánica estadística, claro, solo trate la temperatura como un dato fenomenológico, podemos medirla con un termómetro, ¿a quién le importa lo que es? Pero ahora que tienes la capacidad de hablar sobre cómo β = ( k B T ) 1 trata mucho mejor con "temperaturas negativas" que T hace, todo lo que el estudiante sabe de los cursos anteriores de termodinámica es dolorosamente incompleto y es posible que no desee basar demasiado el material en una buena comprensión de la temperatura y la longitud de onda térmica.

Habiendo dicho todo esto: su enfoque es ciertamente simple y limpio, y podría ser una buena introducción heurística al tema si no desea dedicar una conferencia completa a la ecuación de Sackur-Tetrode.

Gracias por sus comentarios, y es genial tener su perspectiva. Mis estudiantes son estudiantes de ingeniería química y el mayor desafío es ayudarlos a comprender la segunda ley y por qué el calor está relacionado con la entropía (o la aleatoriedad). Usaré el concepto de volumen cuántico, λ 3 para relacionar esto. El único problema que veo en sus comentarios es que necesita entropía para derivar el volumen cuántico. La longitud de onda térmica se puede derivar por Λ = h pag , mi k = pag 2 2 metro , mi k = π k B T . ¿Quiere decir que una de estas ecuaciones requiere S para derivar?
CR Drost: Estuve pensando en este comentario que hiciste: "Te estás perdiendo el término adicional (5/2) kN, que puede ser importante si tienes que trabajar con potenciales químicos". ¿Se refiere al hecho de que no obtengo la definición correcta de potencial químico si omito el término 5/2 para la entropía?
@TedYu Creo que sí. Ciertamente la expresión m = T ( S norte ) mi , V parecería depender crucialmente de ello.
El potencial químico de un gas monoatómico ideal debe ser: m = τ yo norte norte norte q web.mit.edu/ndhillon/www/Teaching/Physics/bookse5.html Curiosamente, obtengo este resultado si omito el término 5/2 con su definición anterior de potencial químico. Por eso me confunde tu comentario. No entiendo por qué necesita el término 5/2 para obtener el potencial químico correcto.
Hice la pregunta aquí si está interesado: physics.stackexchange.com/questions/231080/…
@TedYu: Necesitas ese término porque la expresión que estás derivando es k norte en ( α tu 3 / 2 V norte 5 / 2 ) y la derivada de norte registro norte es registro norte + 1 , por lo que realmente obtienes k en ( α tu 3 / 2 V ) + 5 2 k ( en norte + 1 ) , el extra 5 / 2 k debe equilibrarse con un + 5 / 2 k para obtener el resultado que buscas.
Confío en que tengas razón, pero todavía no puedo superar el hecho de que U es independiente de N. Si U = 3 2 norte k B T , cancelaría toda la dependencia de N dentro del registro en el caso de que sea P constante. Pero supongo que en este caso es E (o U) constante, así que lo entiendo. Todavía perplejo en la otra pregunta sin embargo.